LSAT and Law School Admissions Forum

Get expert LSAT preparation and law school admissions advice from PowerScore Test Preparation.

 Administrator
PowerScore Staff
  • PowerScore Staff
  • Posts: 8916
  • Joined: Feb 02, 2011
|
#47233
Please post your questions below!
 em99
  • Posts: 4
  • Joined: Sep 02, 2018
|
#57073
I was down to answers B and D and ultimately chose D. This question was difficult to me because it seemed like Menkin didn't really have a position within his argument. He was simply pointing out the inconsistency if Hendry were to abide by the principle he stated.

I was too focused on the latter half of Menkin's stimulus, "Then, contrary to what you claim, employee strikes should almost never be permitted." This led me to choosing answer D. However, there was a feeling inside my head that this answer was "too easy" to be correct.

Looking at it now, it appears Menkin does have an opinion. The basis for Menkin to say that strikes should almost never be permitted comes from the principle in Hendry's argument:

Strike harms customers ---> ~Legally permitted

The contrapositive of this principle is that Strikes should be legally permitted ONLY IF if does not harm customers. In this case we can infer that Hendry believes most strikes do not harm customers, while Menkin disagrees.


I guess my question is if you could evaluate my reasoning to come up with the solution. I also want to know how else to eliminate D, since at first glance it was so appealing. Thank you
 Brook Miscoski
PowerScore Staff
  • PowerScore Staff
  • Posts: 418
  • Joined: Sep 13, 2018
|
#58211
Em,

In this case, the question stem asked you what the two disagreed about. It's good that you recognized that Menkin didn't take a position, and that actually helps you pick the right answer.

The difference between (B) and (D) is "harms customers" versus "legally permitted." So, you need to decide whether they disagree about harm to customers, or whether they disagree about whether strikes should be legal.

Henry thinks strikes should be legal--and here is where your observation comes in. Menkin didn't take a position on that issue, he just believed that Henry's reasoning was wrong! Eliminate (D) and pick (B). You spotted all the right tools and just misstepped a little.

Henry states that university strikes are an exception to the rule, because they harm customers. So Henry thinks that most of the time strikes do not harm customers. Menkin claims that Henry is mistake--that harm to customers is the rule, not the exception.
 g_lawyered
  • Posts: 211
  • Joined: Sep 14, 2020
|
#95076
Hi P.S.,
Similar to the question posted above, I also had difficulty understanding their arguments. I interpreted Henry's argument to mean/His conclusion is: that strikes by university faculty's shouldn't be legally permitted because of the premise: "But strikes by university faculty are an exception". An "exception" to the previous sentence which he states "Most employee strikes should be legally permitted. "

Question 1:
This part of the explanation above has gotten me confused:
"Henry states that university strikes are an exception to the rule, because they harm customers. So Henry thinks that most of the time strikes do not harm customers. Menkin claims that Henry is mistake--that harm to customers is the rule, not the exception."

How can we infer that Henry thinks that most of the strikes don't harm customers?
For Henry's principle I understood it to mean: If strike harms customers :arrow: NOT Legally permitted
Contrapositive: Legally permitted :arrow: strike DIDN'T harm customers

Menkin's conclusion is that: If strike harms customer ("if your principle is correct") :arrow: never legally permitted
Contrapositive: legally permitted :arrow: strike DIDN'T harm customer.

Question 2:
In my understanding, they are agreeing about the principle (since Henry's and Menkin's principle translated says the same thing). Because the question asks about what they disagree, I eliminated answer B. What part of the argument tells us that what Henry & Menkin disagree about is answer B?

Question 3:
However, what they disagree about is whether it should/shouldn't be legally permitted- what answer C states. This is the reason why I chose answer C so confidently.
I feel like I misunderstood 1 of the speakers argument and is why I'm stuck on this. Can someone please clarify where I went wrong? :-?

Thanks in advance!
 Robert Carroll
PowerScore Staff
  • PowerScore Staff
  • Posts: 1787
  • Joined: Dec 06, 2013
|
#95098
g_lawyered,

The very conditional you quote for Hendry shows why Hendry believes that most of the time strikes do not harm customers.

As you say:
If strike harms customers :arrow: NOT Legally permitted
Contrapositive: Legally permitted :arrow: strike DIDN'T harm customers
Hendry's first sentence is that most strikes should be legally permitted. Your contrapositive shows that Hendry thinks ALL strikes that should be legally permitted don't harm customers. Therefore, Hendry thinks that most strikes don't harm customers.

Menkin thinks that, according to the same principle, strikes should almost never be legally permitted. So Menkin thinks that, on the contrary, most strikes DO harm customers.

Answer choice (B) is not the principle at all. The principle is the conditional. They disagree on how the facts in reality match or don't match the conditions that would trigger the conditional. Hendry thinks that largely strikes don't harm customers. Menkin thinks that largely strikes do harm customers. So the same conditional, with the background of how often they think strikes harm customers (where they disagree), leads them to very different conclusions.

Menkin makes no mention whatsoever about strikes by university faculty. Answer choice (C) is therefore a non-starter.

Robert Carroll
 g_lawyered
  • Posts: 211
  • Joined: Sep 14, 2020
|
#95105
Hi Robert,
From your explanation I see why C is incorrect answer. But I'm struggling to see systematically how Hendry's principle can be written out conditionally to infer his belief on "most strikes". The premise states "most strikes shouldn't be legally permitted" and yet the principle states "if strike harms employer's customers, not legally permitted".

As you mentioned:
Hendry's first sentence is that most strikes should be legally permitted. Your contrapositive shows that Hendry thinks ALL strikes that should be legally permitted don't harm customers. Therefore, Hendry thinks that most strikes don't harm customers.
The conditional premise states "All strikes harms". My question is: are we to incorporate the premise "most strikes" into the conditional principal "all strikes" and infer that Hendry believes that "most strikes harm customers". I was under the impression that Hendry believed that "all strikes harm customers". This is the disconnect I'm having problems understanding.

Question 2:
Can you please further explain how we can understand Menkin to believe this:
"Menkin thinks that, according to the same principle, strikes should almost never be legally permitted. So Menkin thinks that, on the contrary, most strikes DO harm customers."
Because when I translate what Menkin argument is, I come up with:
"If strike harms customer ("if your principle is correct") :arrow: never legally permitted
Contrapositive: legally permitted :arrow: strike DIDN'T harm customer."
I don't see how we can infer that Menkin believes strikes do harm customers because the principle states if strikes harm customers. I noticed the difference in strong language between Hendry "shouldn't be legally permitted" and Menkin "almost never legally permitted". I think once I understand how we get to infer what Hendry and Menkin's arguments are, I'll be able to understand why B is correct. :-?
Thanks in advance
 Adam Tyson
PowerScore Staff
  • PowerScore Staff
  • Posts: 5153
  • Joined: Apr 14, 2011
|
#95121
Menkin thinks that Hendry is correct when she says that "an employee strike shouldn’t be legally permitted if it would harm the employer’s customers."

Based on that principle, Menkin concludes that "employee strikes should almost never be legally permitted."

How did Menkin get to that conclusion based on that principle? He could only have done so by assuming that almost all employee strikes harm the employer's customers. That is a necessary assumption of his argument. If he did not make that assumption, then his conclusion would make no sense!

Hendry, meanwhile, must believe the opposite, because while she accepts that same principle, she also believes that most strikes should be legally permitted. If she believed that most strikes harm customers, then she could not believe that principle and at the same time believe most strikes should be permitted. Her argument would make no sense. So Hendry must believe that most employee strikes do not harm the customers.

Try approaching both arguments by looking for the assumptions that underlie them. You should find that they rely on opposing assumptions, and that will give you your prephrase and should match answer B!
 g_lawyered
  • Posts: 211
  • Joined: Sep 14, 2020
|
#95138
Hi Adam,
Wow I really missed the mark in solving out what Hendry & Menkin were assuming. I now see why that's significant to lead to correct answer B. :0
1 remaining question for questions with conditional reasoning. I want to make sure I'm understanding the accurate meaning of "most" and "all statements. I know "all" includes conditional reasoning & "most" includes formal logic. But Hendry's argument uses both. Can you please clarify this:
The conditional premise states "All strikes harms". My question is: are we to incorporate the premise "most strikes" into the conditional principal "all strikes" and infer that Hendry believes that "most strikes harm customers". I was under the impression that Hendry believed that "all strikes harm customers". This is the disconnect I'm having problems understanding.
Thanks for all your help!
 Adam Tyson
PowerScore Staff
  • PowerScore Staff
  • Posts: 5153
  • Joined: Apr 14, 2011
|
#95153
Any claim about all (100%) must, by definition, also include claims about most (more than 50%). We sometimes say that "all includes most," So if someone thinks that all strikes are harmful, they must believe that most are harmful. All is greater than 50%, so all IS most!

It does not work the other way, though - someone could believe that most strikes are harmful but not necessarily believe that all are harmful. Most does not require all.

Get the most out of your LSAT Prep Plus subscription.

Analyze and track your performance with our Testing and Analytics Package.